ChaseDream
搜索
123下一页
返回列表 发新帖
查看: 2287|回复: 21
打印 上一主题 下一主题

向好心人问两道逻辑题~~实在是木有看懂题目什么意思

[复制链接]
跳转到指定楼层
楼主
发表于 2012-4-9 18:23:54 | 只看该作者 回帖奖励 |倒序浏览 |阅读模式
1、Editorial:


In Ledland, unemployed adults receive government assistance.To reduce unemployment, the government proposes to supplement the income of those who accept jobs that pay less than government assistance, thus enabling employers to hire workers cheaply.However, the supplement will not raise any worker’s income above what government assistance would provide if he or she were not gainfully employed.Therefore, unemployed people will have no financial incentive to accept jobs that would entitle them to the supplement.



Which of the following, if true about Ledland, most seriously weakens the argument of the editorial?


    A.The government collects no taxes on assistance it provides to unemployed individuals and their families.
    B.Neighboring countries with laws that mandate the minimum wage an employer must pay an employee have higher unemployment rates than Ledland currently has.
    C.People who are employed and look for a new job tend to get higher-paying jobs than job seekers who are unemployed.

    D.The yearly amount unemployed people receive from government assistance is less than the yearly income that the government defines as the poverty level.
    E.People sometimes accept jobs that pay relatively little simply because they enjoy the work.


为啥选C呢?这题目到底是什么意思~~哪位高人指点下?


By competing with rodents for seeds, black ants help control rodent populations that pose a public health risk.However, a very aggressive species of blank ant, the Loma ant, which has recently invaded a certain region, has a venomous sting that is often fatal to humans.Therefore, the planned introduction into that region of ant flies, which prey on Loma ants, would benefit public health.



Which of the following, if true, most strengthens the argument?


A.Ant flies do not attack black ants other than Loma ants.
    B.Loma ants are less effective than many bird species in competing with rodents for seeds.

    C.Certain other species of black ants are more effective than Loma ants in competing with rodents for seeds.
    D.The sting of Loma ants can also be fatal to rodents.
    E.The use of pesticides to control Loma ants could have harmful effects on the environment.
Answer: A
这道题答案是A,但是我认为是B 呀~~我的理解是:
解释:因为要与R争种子,所以黒蚁其实帮助控制了会造成公共健康危机的R的数量。但是,有一种很有攻击性的黑蚂蚁L,最近入侵了某个区域,L有有毒的刺可以让人类致命。因此,计划引进一种ant flies,专吃黑蚂蚁,这样可以有益于公共健康。


这样说的话R和L其实都会对公共健康造成危害,现在要引进某种东西消除L,那么R的数量就会增长,如果要support必须考虑有别的东西也可以控制R的数量,且效果比L要好~~也就是消灭了L 之后要找到L 的替代品。
难道是我理解错了吗?还是答案错了~GWD~~有点不靠谱啊

解释:因为要与R争种子,所以黒蚁其实帮助控制了会造成公共健康危机的R的数量。但是,有一种很有攻击性的黑蚂蚁L,最近入侵了某个区域,L有有毒的刺可以让人类致命。因此,计划引进一种ant flies,专吃黑蚂蚁,这样可以有益于公共健康。


这样说的话R和L其实都会对公共健康造成危害,现在要引进某种东西消除L,那么R的数量就会增长,如果要support必须考虑有别的东西也可以控制R的数量,且效果比L要好~~也就是消灭了L 之后要找到L 的替代品。
难道是我理解错了吗?还是答案错了~GWD~~有点不靠谱啊

收藏收藏 收藏收藏
沙发
发表于 2012-4-9 18:30:24 | 只看该作者
失业的人群呢
拿着政府补助金
现在政府劝失业的人群去找工作
这个工作的工资要是低于最低收入保障
政府就补足差价
要是高于最低收入保障
政府就不管了

结论是没有financial incentive

weaken
C选项说 就业的人再找工作能找到薪水更高的工作 相比没有工作的人 这就是很明显的financial incentive
E选项你高亮了不知道什么意思 但是说的完全不搭边
板凳
发表于 2012-4-9 18:40:18 | 只看该作者
政府为减少失业,提出给予接受工作人群中收入少于政府补助的人补助。但是,此举不会使得该类人群收入高于不工作接受补助的人群。
因此,失业人群没有由于该补助激励去接受工作的financial incentive。就是说政府减少失业的政策目的不能达到。

问削弱,即,由于下列哪个条件的给出,政府可以达到减少失业比率的政策目的。

A 支持政府目的不能达到。
B 邻国--无关
C LS解释挺明白,个人觉得是弱削弱,不是很强烈的削弱,但是相对其他选项看,已经是最优选。
D 改变比较对象,不符题意了。
E 题中没有涉及个人偏好这一说,不好自己徒手添加背景知识的。
地板
发表于 2012-4-9 18:41:55 | 只看该作者
话说我逻辑题目做的少,这个题目貌似是见过= =...能问问是哪里的么?

选项一不小心容易选错的说。。。
5#
发表于 2012-4-9 18:43:09 | 只看该作者
GWD-TN-24
6#
发表于 2012-4-9 18:44:13 | 只看该作者
GWD-TN-24
-- by 会员 lxd19870226 (2012/4/9 18:43:09)


[attachimg=70,140]98738[/attachimg]谢谢。。。。。。o(╯□╰)o啊~~~~~~~~~~~~~~~~

本帖子中包含更多资源

您需要 登录 才可以下载或查看,没有帐号?立即注册

x
7#
发表于 2012-4-9 19:37:11 | 只看该作者
政府为减少失业,提出给予接受工作人群中收入少于政府补助的人补助。但是,此举不会使得该类人群收入高于不工作接受补助的人群。
因此,失业人群没有由于该补助激励去接受工作的financial incentive。就是说政府减少失业的政策目的不能达到。

问削弱,即,由于下列哪个条件的给出,政府可以达到减少失业比率的政策目的。

A 支持政府目的不能达到。
B 邻国--无关
C LS解释挺明白,个人觉得是弱削弱,不是很强烈的削弱,但是相对其他选项看,已经是最优选。
D 改变比较对象,不符题意了。
E 题中没有涉及个人偏好这一说,不好自己徒手添加背景知识的。
-- by 会员 泾渭不凡 (2012/4/9 18:40:18)

你全天活动在CD?
8#
发表于 2012-4-9 21:34:58 | 只看该作者
政府为减少失业,提出给予接受工作人群中收入少于政府补助的人补助。但是,此举不会使得该类人群收入高于不工作接受补助的人群。
因此,失业人群没有由于该补助激励去接受工作的financial incentive。就是说政府减少失业的政策目的不能达到。

问削弱,即,由于下列哪个条件的给出,政府可以达到减少失业比率的政策目的。

A 支持政府目的不能达到。
B 邻国--无关
C LS解释挺明白,个人觉得是弱削弱,不是很强烈的削弱,但是相对其他选项看,已经是最优选。
D 改变比较对象,不符题意了。
E 题中没有涉及个人偏好这一说,不好自己徒手添加背景知识的。
-- by 会员 泾渭不凡 (2012/4/9 18:40:18)


你全天活动在CD?
-- by 会员 Elroy (2012/4/9 19:37:11)


没那么敬业哈,就是弄其他的东西要联网的,CD界面就开着,定时刷一下看有没有狗狗哈。

同学加油。
9#
 楼主| 发表于 2012-4-10 00:13:49 | 只看该作者
However, the supplement will not raise any worker’s income above what government assistance would provide if he or she were not gainfully employed.这句话的意思,也就是说政府会给那些工作了的,但是收入还没有不工作领政府补贴的人多的人补贴是吧?~~政府给补贴,本来是想让雇主hire 员工更便宜,这样薪水就平衡了~~但是也最多就达到没工作的人的补助水平了~~于是就不会给不工作的人一个financial incentive~~~(是不?)如果我理解正确的话要谢谢大牛你啊!!如果理解不正确请指正哦~
10#
 楼主| 发表于 2012-4-10 00:15:11 | 只看该作者
亲你头像很有feel啊~~谢谢你的解答哦!很耐心!灰常感谢!

可以有空帮我看下第二题么~~我把我的思考过程写在上面了~你看看是我的问题,还是答案本来就错了~
您需要登录后才可以回帖 登录 | 立即注册

Mark一下! 看一下! 顶楼主! 感谢分享! 快速回复:

手机版|ChaseDream|GMT+8, 2025-9-4 05:18
京公网安备11010202008513号 京ICP证101109号 京ICP备12012021号

ChaseDream 论坛

© 2003-2025 ChaseDream.com. All Rights Reserved.

返回顶部